Quantcast
  • Register
PhysicsOverflow is a next-generation academic platform for physicists and astronomers, including a community peer review system and a postgraduate-level discussion forum analogous to MathOverflow.

Welcome to PhysicsOverflow! PhysicsOverflow is an open platform for community peer review and graduate-level Physics discussion.

Please help promote PhysicsOverflow ads elsewhere if you like it.

News

PO is now at the Physics Department of Bielefeld University!

New printer friendly PO pages!

Migration to Bielefeld University was successful!

Please vote for this year's PhysicsOverflow ads!

Please do help out in categorising submissions. Submit a paper to PhysicsOverflow!

... see more

Tools for paper authors

Submit paper
Claim Paper Authorship

Tools for SE users

Search User
Reclaim SE Account
Request Account Merger
Nativise imported posts
Claim post (deleted users)
Import SE post

Users whose questions have been imported from Physics Stack Exchange, Theoretical Physics Stack Exchange, or any other Stack Exchange site are kindly requested to reclaim their account and not to register as a new user.

Public \(\beta\) tools

Report a bug with a feature
Request a new functionality
404 page design
Send feedback

Attributions

(propose a free ad)

Site Statistics

205 submissions , 163 unreviewed
5,047 questions , 2,200 unanswered
5,345 answers , 22,709 comments
1,470 users with positive rep
816 active unimported users
More ...

  Locality in the scattering amplitude

+ 7 like - 0 dislike
1791 views

Early in this talk by Nima Arkani-Hamed, he describes what locality means for scattering amplitudes. "Locality tells you that the only poles in the scattering amplitude occur when the sum of a subset of the momenta of the particles goes on shell."

The talk then goes on to describe his recent research, but right now I'm not worrying about understanding that. I'm just trying to understand the statement quoted above. I know what locality looks like in the Lagrangian density (the terms of the Lagrangian density contain field operators which are all functions of the same spacetime coordinate), but it's not at all obvious to me how that is equivalent to the above statement about scattering amplitudes.

Can you explain it (preferably at about the level of a first graduate course in QFT)?

This post imported from StackExchange Physics at 2014-04-24 02:32 (UCT), posted by SE-user Tim Goodman
asked Sep 22, 2013 in Theoretical Physics by Tim Goodman (45 points) [ no revision ]
I don't have an answer but I think it is related to LSZ reduction, hope this might help.

This post imported from StackExchange Physics at 2014-04-24 02:32 (UCT), posted by SE-user Jia Yiyang
Just thinking out loud -- I wonder if this is related to a statement he made in his last paper on the subject: physics.stackexchange.com/questions/54354/…

This post imported from StackExchange Physics at 2014-04-24 02:32 (UCT), posted by SE-user Siva
I guess by now I should know that Weinberg's QFT books have everything and more! :-) Check Section 10.2 (Book1), on Polology.

This post imported from StackExchange Physics at 2014-04-24 02:32 (UCT), posted by SE-user Siva

2 Answers

+ 3 like - 0 dislike

It is not an answer, but just some hints.

Consider the simplest free QFT with a massless bosonic scalar, the terms in the Lagrangian are local : $\phi(x) \square \phi(x)$. Considering an interacting theory ($\phi^3, \phi^4$). You are interested in calculate scattering amplitudes with incoming particles and outcoming particles. You will use the propagator in $\frac{1}{k^2}$ which form is direcly linked to the above Lagrangian term, and you may note that this propagator has a pole when $k$ is on-shell.

If you consider only a tree-level diagram, the transition amplitude is simply the product of propagators, each propagator could be written $\frac{1}{l^2}$, where $l$ is the sum of some external momenta (at each vertex, you have momentum conservation). So a pole of the scattering amplitude corresponds to the pole of the propagators, and this corresponds by putting on-shell some particular sum of the external momenta.

Now, consider a loop-diagram, with dimensional regularization, like $I(q) \sim g^2 (\mu^2)^\epsilon\int d^{4-\epsilon}p \frac{1}{p^2}\frac{1}{(p-q)^2}$,where $\epsilon$ is $>0$, $q$ is an external momentum. By using the Feynmann formula $\frac{1}{ab} = \int_0^1 \frac{dz}{[az+b(1-z)]^2}$, you will get : $I(q) \sim g^2 (\mu^2)^\epsilon\int_0^1 dz \int (d^{4-\epsilon}p) \dfrac{1}{[p^2 - 2p.q(1-z)+q^2(1-z)]^2}$, and finally :

$I(q) \sim g^2 (\mu^2)^\epsilon ~\Gamma(\frac{\epsilon}{2})\int_0^1 dz \dfrac{1}{[q^2 z(1-z) ]^{\large \frac{\epsilon}{2}}}$

Here, $\epsilon$ is $>0$, so we see that if $q^2=0$, the integral is not defined, so $q^2=0$ should represent a pole for the scattering amplitude.

The relation with the locality could be seen as looking at the Fourier transform (taking $\epsilon=0$) of the scattering amplitude which could be written $I(x) \sim [D(x)]^2$, where $D(x)$ is the propagator in space-time coordinates.

Now, we should hope that any scattering amplitude, with loops, should have poles, which corresponds to some particular sum of the external momenta being on-shell.

This post imported from StackExchange Physics at 2014-04-24 02:32 (UCT), posted by SE-user Trimok
answered Sep 23, 2013 by Trimok (955 points) [ no revision ]
It looks along the right lines to me. You can get similar conclusions from the standpoint of the old analytic S matrix theory: there the causality (which translate to locality) conditions are encoded in the analytic structure of the S matrix, which includes a simple pole for single particle exchange, two poles linked by a branch cut for two particle exchange etc. The simple pole forces the exchanged particle on shell. The multiple poles force a consistent network of momentum exchanges, a bit like Kirchoff's laws.

This post imported from StackExchange Physics at 2014-04-24 02:32 (UCT), posted by SE-user twistor59
It's clear that propagators going on-shell will contribute poles to the S-matrix. I feel the non-trivial part is that these are the only possible poles. @Trimok's argument is suggestive, but does not quite seem watertight to me, especially when the theory might be non-perturbative.

This post imported from StackExchange Physics at 2014-04-24 02:32 (UCT), posted by SE-user Siva
@twistor59, Could you suggest a readable introduction to S-matrix methods where I might find an explanation of your statement? The connections to Kirchoff's laws seem particularly intriguing :-) Ps: By "readable" I mean language and notation that is not particularly arcane and reasonably in sync with modern conventions.

This post imported from StackExchange Physics at 2014-04-24 02:32 (UCT), posted by SE-user Siva
@Siva I was thinking along the lines of the discussion in Bjorken and Drell chapter 18.

This post imported from StackExchange Physics at 2014-04-24 02:32 (UCT), posted by SE-user twistor59
+ 1 like - 0 dislike

Here's the sketch of an attempted explanation, and a work in progress.

Unitarity implies the optical theorem (Ref: Peskin & Schroeder Section 7.3), which says

$$\Im[\mathcal{M}_{i \rightarrow f}] = \sum_{\textrm{m=middle}} \int d\Pi_{\textrm{m}} \mathcal{M}(i\rightarrow m) \mathcal{M}^* (m \rightarrow f)$$ Essentially, you're re-expressing the scattering amplitude $\mathcal{M}(i \rightarrow f)$ as a sum over possible channels, and adding up the residues (of poles) whenever those intermediate states go on-shell. (NB: Since the matrix element is an analytic function of the variables, one should be able to obtain the real part given the imaginary part)

To my understanding, in section 10.2 (Polology) of his QFT-Book1 Weinberg shows that this kind of interpretation (poles when there's some on-shell intermediate state aka a "resonance") holds even when the intermediate states $m$ are bound states (non-perturbative in general) and not necessarily just degrees of freedom in the free theory.

It is the same idea behind the spectral representation of the propagator (a la Kallen-Lehmann. Refs: Peskin & Schroeder Section 7.1, Weinberg QFT1 Section 10.7)

The exact step where/why locality comes in is not yet clear to me.

Ps: I would appreciate help in fleshing out the details of this answer -- maybe making it a community wiki.

This post imported from StackExchange Physics at 2014-04-24 02:32 (UCT), posted by SE-user Siva
answered Sep 23, 2013 by Siva (720 points) [ no revision ]
Regarding locality and the lagrangian density, I might not want to depend on that formulation (of locality), since such a Lagrangian formulation of the theory (which assumes a preferred basis for the degrees of freedom) might only make sense when you're perturbing around the free theory and using the states of the free theory as your Fock space basis. I think the claim/result is more general than that.

This post imported from StackExchange Physics at 2014-04-24 02:32 (UCT), posted by SE-user Siva
As said in the different Nima Arkani-Hamed talks, locality (or causality) dictates the poles, and unitarity dictates the behaviour at the poles, that is factorization (as, in the optical theorem).

This post imported from StackExchange Physics at 2014-04-24 02:32 (UCT), posted by SE-user Trimok

Your answer

Please use answers only to (at least partly) answer questions. To comment, discuss, or ask for clarification, leave a comment instead.
To mask links under text, please type your text, highlight it, and click the "link" button. You can then enter your link URL.
Please consult the FAQ for as to how to format your post.
This is the answer box; if you want to write a comment instead, please use the 'add comment' button.
Live preview (may slow down editor)   Preview
Your name to display (optional):
Privacy: Your email address will only be used for sending these notifications.
Anti-spam verification:
If you are a human please identify the position of the character covered by the symbol $\varnothing$ in the following word:
p$\hbar$ysicsOverflo$\varnothing$
Then drag the red bullet below over the corresponding character of our banner. When you drop it there, the bullet changes to green (on slow internet connections after a few seconds).
Please complete the anti-spam verification




user contributions licensed under cc by-sa 3.0 with attribution required

Your rights
...